Compacité

jm14d
Modifié (September 2022) dans Topologie
Bonjour, je considère, dans R muni de sa topologie classique, l'ensemble des rationnels éléments de [0,1] : est-ce un compact ?
La réponse est sûrement "non", les compacts de R étant les intervalles fermés et bornés, et les réunions de tels intervalles.
Mais j'ai du mal à voir pourquoi, en revenant à la définition "de tout recouvrement ouvert on peut extraire un recouvrement fini".
Quel serait un recouvrement ouvert de cet ensemble, dont on ne pourrait pas extraire un recouvrement fini ?

Réponses

  • stfj
    Modifié (September 2022)
    Il y a des suites de rationnels de $[0,1]$ qui convergent vers des irrationnels. Donc $[0,1] \cap \mathbb{Q}$ n'est pas compact : il n'est pas fermé. Donc, théoriquement, il existe effectivement un recouvrement ouvert de $[0,1] \cap \mathbb{Q}$ dont on ne peut extraire un sous-recouvrement fini. Mais je ne vois pas de construction effective. De toutes façons, les caractérisations par les suites sont des outils puissants : la preuve.
  • troisqua
    Modifié (September 2022)
    Les ouverts $]n+\sqrt{2};n+1+\sqrt{2}[$,  pour $n$ parcourant les relatifs, recouvrent $\Q$ mais il n'y a pas de sous-recouvrement fini (sinon $\Q$ serait borné).
  • JLapin
    Modifié (September 2022)
    jm14d a dit :
    La réponse est sûrement "non", les compacts de R étant les intervalles fermés et bornés, et les réunions de tels intervalles.
    Attention, toute partie de $\R$ et pas seulement les compacts est réunion de singletons donc d'intervalles fermés et bornés...
  • La phrase correcte étant "les compacts de $\mathbb R$ sont les fermés bornés". Une réunion de fermés n'étant en général pas fermée...

    Il n'y a pas que le caractère non borné de $\mathbb Q$ qui est en cause dans sa non compacité. Voici un recouvrement ouvert de $\mathbb Q \cap ]0, 1[$ n'admettant pas de sous-recouvrement fini : soit $x$ un irrationnel dans $[0, 1]$, alors on a $\mathbb Q \cap ]0, 1[ \subset \bigcup_{n \geq 1} \left ]0, x - \frac{1}{n}\right[ \cup \left] x + \frac{1}{n}, 1\right[$.
  • Et voici un recouvrement ouvert de $\mathbb Q \cap [0, 1]$ n'admettant pas de sous-recouvrement fini : soit $x$ un irrationnel dans $[0,1]$, alors on a $\mathbb Q \cap [0, 1] \subset \bigcup_{n \geq 1} \left ]-\infty, x - \frac{1}{n}\right[ \cup \left] x + \frac{1}{n}, +\infty\right[$.
  • Lars
    Modifié (September 2022)
    Bonjour
    On peut aussi montrer que si c'est vrai alors tout irrationnel est rationnel (ie qu'il n'existe pas de nombre irrationnel) en utilisant une fonction continue sur un compact à valeurs réelles atteint ses bornes.
  • Merci beaucoup, vos réponses m'ont bien aidé, et montrent aussi qu'on a pas mal d'angles différents sous lesquels aborder les questions de compacité. Bon début de semaine à tous et toutes !
Connectez-vous ou Inscrivez-vous pour répondre.